Show that for positive numbers a,b,c,d, $sum_cyc ab leq frac14left(sum_cyc a right)^2$ and … [duplicate]

The name of the pictureThe name of the pictureThe name of the pictureClash Royale CLAN TAG#URR8PPP











up vote
1
down vote

favorite













This question already has an answer here:



  • Showing $sumlimits_cycablefrac14left(sumlimits_cycaright)^2$

    2 answers



Let a,b,c,d be four positive real numbers. Show that
$$sum_cyc ab leq frac14left(sum_cyc a right)^2$$
and
$$sum_cyc abc leq frac116left(sum_cyc a right)^3$$
My textbook said these inequalities can be referred from the AM-GM inequality but didn't say how to derive it. I can find only looser bounds.
That is, for each term in the left hand side of the first inequality, by AM-GM inequality,
$$ab leq left(fraca+b2right)^2 leq frac14 left(a+b+c+dright)^2$$
$$Longrightarrow sum_cyc ab leq frac44 left(sum_cyc a right)^2$$
Using the same techniques, we get another loose bound for $sum_cyc abc$.
How could I get the bounds in the original inequalities? Please suggest.



PS. It seems like if these inequalities are true, we may get a general relationship for n positive variables as $sum_cyc a_1 ... a_k leq frac1n^k-1left(sum_cyc a_i right)^k$.



PS2. Sorry for double posting. There was an old thread here: Showing $sumlimits_cycablefrac14left(sumlimits_cycaright)^2$ One answer points to Maclaurin's inequality, but I still cannot figure out how to prove it using AM-GM.










share|cite|improve this question















marked as duplicate by Michael Rozenberg inequality
Users with the  inequality badge can single-handedly close inequality questions as duplicates and reopen them as needed.

StackExchange.ready(function()
if (StackExchange.options.isMobile) return;

$('.dupe-hammer-message-hover:not(.hover-bound)').each(function()
var $hover = $(this).addClass('hover-bound'),
$msg = $hover.siblings('.dupe-hammer-message');

$hover.hover(
function()
$hover.showInfoMessage('',
messageElement: $msg.clone().show(),
transient: false,
position: my: 'bottom left', at: 'top center', offsetTop: -7 ,
dismissable: false,
relativeToBody: true
);
,
function()
StackExchange.helpers.removeMessages();

);
);
);
Sep 9 at 13:12


This question has been asked before and already has an answer. If those answers do not fully address your question, please ask a new question.


















    up vote
    1
    down vote

    favorite













    This question already has an answer here:



    • Showing $sumlimits_cycablefrac14left(sumlimits_cycaright)^2$

      2 answers



    Let a,b,c,d be four positive real numbers. Show that
    $$sum_cyc ab leq frac14left(sum_cyc a right)^2$$
    and
    $$sum_cyc abc leq frac116left(sum_cyc a right)^3$$
    My textbook said these inequalities can be referred from the AM-GM inequality but didn't say how to derive it. I can find only looser bounds.
    That is, for each term in the left hand side of the first inequality, by AM-GM inequality,
    $$ab leq left(fraca+b2right)^2 leq frac14 left(a+b+c+dright)^2$$
    $$Longrightarrow sum_cyc ab leq frac44 left(sum_cyc a right)^2$$
    Using the same techniques, we get another loose bound for $sum_cyc abc$.
    How could I get the bounds in the original inequalities? Please suggest.



    PS. It seems like if these inequalities are true, we may get a general relationship for n positive variables as $sum_cyc a_1 ... a_k leq frac1n^k-1left(sum_cyc a_i right)^k$.



    PS2. Sorry for double posting. There was an old thread here: Showing $sumlimits_cycablefrac14left(sumlimits_cycaright)^2$ One answer points to Maclaurin's inequality, but I still cannot figure out how to prove it using AM-GM.










    share|cite|improve this question















    marked as duplicate by Michael Rozenberg inequality
    Users with the  inequality badge can single-handedly close inequality questions as duplicates and reopen them as needed.

    StackExchange.ready(function()
    if (StackExchange.options.isMobile) return;

    $('.dupe-hammer-message-hover:not(.hover-bound)').each(function()
    var $hover = $(this).addClass('hover-bound'),
    $msg = $hover.siblings('.dupe-hammer-message');

    $hover.hover(
    function()
    $hover.showInfoMessage('',
    messageElement: $msg.clone().show(),
    transient: false,
    position: my: 'bottom left', at: 'top center', offsetTop: -7 ,
    dismissable: false,
    relativeToBody: true
    );
    ,
    function()
    StackExchange.helpers.removeMessages();

    );
    );
    );
    Sep 9 at 13:12


    This question has been asked before and already has an answer. If those answers do not fully address your question, please ask a new question.
















      up vote
      1
      down vote

      favorite









      up vote
      1
      down vote

      favorite












      This question already has an answer here:



      • Showing $sumlimits_cycablefrac14left(sumlimits_cycaright)^2$

        2 answers



      Let a,b,c,d be four positive real numbers. Show that
      $$sum_cyc ab leq frac14left(sum_cyc a right)^2$$
      and
      $$sum_cyc abc leq frac116left(sum_cyc a right)^3$$
      My textbook said these inequalities can be referred from the AM-GM inequality but didn't say how to derive it. I can find only looser bounds.
      That is, for each term in the left hand side of the first inequality, by AM-GM inequality,
      $$ab leq left(fraca+b2right)^2 leq frac14 left(a+b+c+dright)^2$$
      $$Longrightarrow sum_cyc ab leq frac44 left(sum_cyc a right)^2$$
      Using the same techniques, we get another loose bound for $sum_cyc abc$.
      How could I get the bounds in the original inequalities? Please suggest.



      PS. It seems like if these inequalities are true, we may get a general relationship for n positive variables as $sum_cyc a_1 ... a_k leq frac1n^k-1left(sum_cyc a_i right)^k$.



      PS2. Sorry for double posting. There was an old thread here: Showing $sumlimits_cycablefrac14left(sumlimits_cycaright)^2$ One answer points to Maclaurin's inequality, but I still cannot figure out how to prove it using AM-GM.










      share|cite|improve this question
















      This question already has an answer here:



      • Showing $sumlimits_cycablefrac14left(sumlimits_cycaright)^2$

        2 answers



      Let a,b,c,d be four positive real numbers. Show that
      $$sum_cyc ab leq frac14left(sum_cyc a right)^2$$
      and
      $$sum_cyc abc leq frac116left(sum_cyc a right)^3$$
      My textbook said these inequalities can be referred from the AM-GM inequality but didn't say how to derive it. I can find only looser bounds.
      That is, for each term in the left hand side of the first inequality, by AM-GM inequality,
      $$ab leq left(fraca+b2right)^2 leq frac14 left(a+b+c+dright)^2$$
      $$Longrightarrow sum_cyc ab leq frac44 left(sum_cyc a right)^2$$
      Using the same techniques, we get another loose bound for $sum_cyc abc$.
      How could I get the bounds in the original inequalities? Please suggest.



      PS. It seems like if these inequalities are true, we may get a general relationship for n positive variables as $sum_cyc a_1 ... a_k leq frac1n^k-1left(sum_cyc a_i right)^k$.



      PS2. Sorry for double posting. There was an old thread here: Showing $sumlimits_cycablefrac14left(sumlimits_cycaright)^2$ One answer points to Maclaurin's inequality, but I still cannot figure out how to prove it using AM-GM.





      This question already has an answer here:



      • Showing $sumlimits_cycablefrac14left(sumlimits_cycaright)^2$

        2 answers







      inequality summation symmetric-polynomials a.m.-g.m.-inequality






      share|cite|improve this question















      share|cite|improve this question













      share|cite|improve this question




      share|cite|improve this question








      edited Sep 9 at 14:47

























      asked Sep 9 at 12:25









      DKSG

      163




      163




      marked as duplicate by Michael Rozenberg inequality
      Users with the  inequality badge can single-handedly close inequality questions as duplicates and reopen them as needed.

      StackExchange.ready(function()
      if (StackExchange.options.isMobile) return;

      $('.dupe-hammer-message-hover:not(.hover-bound)').each(function()
      var $hover = $(this).addClass('hover-bound'),
      $msg = $hover.siblings('.dupe-hammer-message');

      $hover.hover(
      function()
      $hover.showInfoMessage('',
      messageElement: $msg.clone().show(),
      transient: false,
      position: my: 'bottom left', at: 'top center', offsetTop: -7 ,
      dismissable: false,
      relativeToBody: true
      );
      ,
      function()
      StackExchange.helpers.removeMessages();

      );
      );
      );
      Sep 9 at 13:12


      This question has been asked before and already has an answer. If those answers do not fully address your question, please ask a new question.






      marked as duplicate by Michael Rozenberg inequality
      Users with the  inequality badge can single-handedly close inequality questions as duplicates and reopen them as needed.

      StackExchange.ready(function()
      if (StackExchange.options.isMobile) return;

      $('.dupe-hammer-message-hover:not(.hover-bound)').each(function()
      var $hover = $(this).addClass('hover-bound'),
      $msg = $hover.siblings('.dupe-hammer-message');

      $hover.hover(
      function()
      $hover.showInfoMessage('',
      messageElement: $msg.clone().show(),
      transient: false,
      position: my: 'bottom left', at: 'top center', offsetTop: -7 ,
      dismissable: false,
      relativeToBody: true
      );
      ,
      function()
      StackExchange.helpers.removeMessages();

      );
      );
      );
      Sep 9 at 13:12


      This question has been asked before and already has an answer. If those answers do not fully address your question, please ask a new question.






















          1 Answer
          1






          active

          oldest

          votes

















          up vote
          1
          down vote













          The first inequality.



          By AM-GM
          $$sum_cycab=(a+c)(b+d)leqleft(fraca+b+c+d2right)^2=frac14(a+b+c+d)^2.$$
          I used that
          $$sum_cycab=ab+bc+cd+da=b(a+c)+d(a+c)=(a+c)(b+d).$$
          The second inequality follows from Maclaurin.



          Let $a+b+c+d=4u$, $ab+ac+bc+ad+bd+cd=6v^2,$ where $v>0$ and $abc+abd+acd+bcd=4w^3$.



          Thus, by Maclaurin $$ugeq vgeq w.$$



          About Maclaurin see here: https://en.wikipedia.org/wiki/Maclaurin%27s_inequality



          Your last inequality is wrong.



          Try $n=5$ and $k=2$.



          But for $n=5$ and $k=3$ it's true.






          share|cite|improve this answer






















          • Can you explain more on the left hand side? There are only 4 terms in your $sum_cyc ab$, but in the questions, there are 6 terms (i.e., ab + bc + ac + cd + ad + bd).
            – DKSG
            Sep 9 at 12:44











          • @DKSG I added something. See now.
            – Michael Rozenberg
            Sep 9 at 12:47










          • Thank you but I still don't understand why your cyclic sum $sum_cyc ab$ expansion has only 4 terms. It should have 6 terms, shouldn't it?
            – DKSG
            Sep 9 at 13:00











          • @DKSG No. It should be four terms. $ab+ac+ad+bc+bd+cd=frac14sumlimits_symab.$ Cyclic it's $arightarrow brightarrow crightarrow drightarrow a$. (Four times "$rightarrow$" give four terms).
            – Michael Rozenberg
            Sep 9 at 13:02











          • By the way, there is a very nice proof of the last inequality for $n=5$ and $k=3$. If you want to see a solution open another topic. This topic I'll close because it's duplicate.
            – Michael Rozenberg
            Sep 9 at 13:09


















          1 Answer
          1






          active

          oldest

          votes








          1 Answer
          1






          active

          oldest

          votes









          active

          oldest

          votes






          active

          oldest

          votes








          up vote
          1
          down vote













          The first inequality.



          By AM-GM
          $$sum_cycab=(a+c)(b+d)leqleft(fraca+b+c+d2right)^2=frac14(a+b+c+d)^2.$$
          I used that
          $$sum_cycab=ab+bc+cd+da=b(a+c)+d(a+c)=(a+c)(b+d).$$
          The second inequality follows from Maclaurin.



          Let $a+b+c+d=4u$, $ab+ac+bc+ad+bd+cd=6v^2,$ where $v>0$ and $abc+abd+acd+bcd=4w^3$.



          Thus, by Maclaurin $$ugeq vgeq w.$$



          About Maclaurin see here: https://en.wikipedia.org/wiki/Maclaurin%27s_inequality



          Your last inequality is wrong.



          Try $n=5$ and $k=2$.



          But for $n=5$ and $k=3$ it's true.






          share|cite|improve this answer






















          • Can you explain more on the left hand side? There are only 4 terms in your $sum_cyc ab$, but in the questions, there are 6 terms (i.e., ab + bc + ac + cd + ad + bd).
            – DKSG
            Sep 9 at 12:44











          • @DKSG I added something. See now.
            – Michael Rozenberg
            Sep 9 at 12:47










          • Thank you but I still don't understand why your cyclic sum $sum_cyc ab$ expansion has only 4 terms. It should have 6 terms, shouldn't it?
            – DKSG
            Sep 9 at 13:00











          • @DKSG No. It should be four terms. $ab+ac+ad+bc+bd+cd=frac14sumlimits_symab.$ Cyclic it's $arightarrow brightarrow crightarrow drightarrow a$. (Four times "$rightarrow$" give four terms).
            – Michael Rozenberg
            Sep 9 at 13:02











          • By the way, there is a very nice proof of the last inequality for $n=5$ and $k=3$. If you want to see a solution open another topic. This topic I'll close because it's duplicate.
            – Michael Rozenberg
            Sep 9 at 13:09















          up vote
          1
          down vote













          The first inequality.



          By AM-GM
          $$sum_cycab=(a+c)(b+d)leqleft(fraca+b+c+d2right)^2=frac14(a+b+c+d)^2.$$
          I used that
          $$sum_cycab=ab+bc+cd+da=b(a+c)+d(a+c)=(a+c)(b+d).$$
          The second inequality follows from Maclaurin.



          Let $a+b+c+d=4u$, $ab+ac+bc+ad+bd+cd=6v^2,$ where $v>0$ and $abc+abd+acd+bcd=4w^3$.



          Thus, by Maclaurin $$ugeq vgeq w.$$



          About Maclaurin see here: https://en.wikipedia.org/wiki/Maclaurin%27s_inequality



          Your last inequality is wrong.



          Try $n=5$ and $k=2$.



          But for $n=5$ and $k=3$ it's true.






          share|cite|improve this answer






















          • Can you explain more on the left hand side? There are only 4 terms in your $sum_cyc ab$, but in the questions, there are 6 terms (i.e., ab + bc + ac + cd + ad + bd).
            – DKSG
            Sep 9 at 12:44











          • @DKSG I added something. See now.
            – Michael Rozenberg
            Sep 9 at 12:47










          • Thank you but I still don't understand why your cyclic sum $sum_cyc ab$ expansion has only 4 terms. It should have 6 terms, shouldn't it?
            – DKSG
            Sep 9 at 13:00











          • @DKSG No. It should be four terms. $ab+ac+ad+bc+bd+cd=frac14sumlimits_symab.$ Cyclic it's $arightarrow brightarrow crightarrow drightarrow a$. (Four times "$rightarrow$" give four terms).
            – Michael Rozenberg
            Sep 9 at 13:02











          • By the way, there is a very nice proof of the last inequality for $n=5$ and $k=3$. If you want to see a solution open another topic. This topic I'll close because it's duplicate.
            – Michael Rozenberg
            Sep 9 at 13:09













          up vote
          1
          down vote










          up vote
          1
          down vote









          The first inequality.



          By AM-GM
          $$sum_cycab=(a+c)(b+d)leqleft(fraca+b+c+d2right)^2=frac14(a+b+c+d)^2.$$
          I used that
          $$sum_cycab=ab+bc+cd+da=b(a+c)+d(a+c)=(a+c)(b+d).$$
          The second inequality follows from Maclaurin.



          Let $a+b+c+d=4u$, $ab+ac+bc+ad+bd+cd=6v^2,$ where $v>0$ and $abc+abd+acd+bcd=4w^3$.



          Thus, by Maclaurin $$ugeq vgeq w.$$



          About Maclaurin see here: https://en.wikipedia.org/wiki/Maclaurin%27s_inequality



          Your last inequality is wrong.



          Try $n=5$ and $k=2$.



          But for $n=5$ and $k=3$ it's true.






          share|cite|improve this answer














          The first inequality.



          By AM-GM
          $$sum_cycab=(a+c)(b+d)leqleft(fraca+b+c+d2right)^2=frac14(a+b+c+d)^2.$$
          I used that
          $$sum_cycab=ab+bc+cd+da=b(a+c)+d(a+c)=(a+c)(b+d).$$
          The second inequality follows from Maclaurin.



          Let $a+b+c+d=4u$, $ab+ac+bc+ad+bd+cd=6v^2,$ where $v>0$ and $abc+abd+acd+bcd=4w^3$.



          Thus, by Maclaurin $$ugeq vgeq w.$$



          About Maclaurin see here: https://en.wikipedia.org/wiki/Maclaurin%27s_inequality



          Your last inequality is wrong.



          Try $n=5$ and $k=2$.



          But for $n=5$ and $k=3$ it's true.







          share|cite|improve this answer














          share|cite|improve this answer



          share|cite|improve this answer








          edited Sep 9 at 12:55

























          answered Sep 9 at 12:42









          Michael Rozenberg

          89.3k1582179




          89.3k1582179











          • Can you explain more on the left hand side? There are only 4 terms in your $sum_cyc ab$, but in the questions, there are 6 terms (i.e., ab + bc + ac + cd + ad + bd).
            – DKSG
            Sep 9 at 12:44











          • @DKSG I added something. See now.
            – Michael Rozenberg
            Sep 9 at 12:47










          • Thank you but I still don't understand why your cyclic sum $sum_cyc ab$ expansion has only 4 terms. It should have 6 terms, shouldn't it?
            – DKSG
            Sep 9 at 13:00











          • @DKSG No. It should be four terms. $ab+ac+ad+bc+bd+cd=frac14sumlimits_symab.$ Cyclic it's $arightarrow brightarrow crightarrow drightarrow a$. (Four times "$rightarrow$" give four terms).
            – Michael Rozenberg
            Sep 9 at 13:02











          • By the way, there is a very nice proof of the last inequality for $n=5$ and $k=3$. If you want to see a solution open another topic. This topic I'll close because it's duplicate.
            – Michael Rozenberg
            Sep 9 at 13:09

















          • Can you explain more on the left hand side? There are only 4 terms in your $sum_cyc ab$, but in the questions, there are 6 terms (i.e., ab + bc + ac + cd + ad + bd).
            – DKSG
            Sep 9 at 12:44











          • @DKSG I added something. See now.
            – Michael Rozenberg
            Sep 9 at 12:47










          • Thank you but I still don't understand why your cyclic sum $sum_cyc ab$ expansion has only 4 terms. It should have 6 terms, shouldn't it?
            – DKSG
            Sep 9 at 13:00











          • @DKSG No. It should be four terms. $ab+ac+ad+bc+bd+cd=frac14sumlimits_symab.$ Cyclic it's $arightarrow brightarrow crightarrow drightarrow a$. (Four times "$rightarrow$" give four terms).
            – Michael Rozenberg
            Sep 9 at 13:02











          • By the way, there is a very nice proof of the last inequality for $n=5$ and $k=3$. If you want to see a solution open another topic. This topic I'll close because it's duplicate.
            – Michael Rozenberg
            Sep 9 at 13:09
















          Can you explain more on the left hand side? There are only 4 terms in your $sum_cyc ab$, but in the questions, there are 6 terms (i.e., ab + bc + ac + cd + ad + bd).
          – DKSG
          Sep 9 at 12:44





          Can you explain more on the left hand side? There are only 4 terms in your $sum_cyc ab$, but in the questions, there are 6 terms (i.e., ab + bc + ac + cd + ad + bd).
          – DKSG
          Sep 9 at 12:44













          @DKSG I added something. See now.
          – Michael Rozenberg
          Sep 9 at 12:47




          @DKSG I added something. See now.
          – Michael Rozenberg
          Sep 9 at 12:47












          Thank you but I still don't understand why your cyclic sum $sum_cyc ab$ expansion has only 4 terms. It should have 6 terms, shouldn't it?
          – DKSG
          Sep 9 at 13:00





          Thank you but I still don't understand why your cyclic sum $sum_cyc ab$ expansion has only 4 terms. It should have 6 terms, shouldn't it?
          – DKSG
          Sep 9 at 13:00













          @DKSG No. It should be four terms. $ab+ac+ad+bc+bd+cd=frac14sumlimits_symab.$ Cyclic it's $arightarrow brightarrow crightarrow drightarrow a$. (Four times "$rightarrow$" give four terms).
          – Michael Rozenberg
          Sep 9 at 13:02





          @DKSG No. It should be four terms. $ab+ac+ad+bc+bd+cd=frac14sumlimits_symab.$ Cyclic it's $arightarrow brightarrow crightarrow drightarrow a$. (Four times "$rightarrow$" give four terms).
          – Michael Rozenberg
          Sep 9 at 13:02













          By the way, there is a very nice proof of the last inequality for $n=5$ and $k=3$. If you want to see a solution open another topic. This topic I'll close because it's duplicate.
          – Michael Rozenberg
          Sep 9 at 13:09





          By the way, there is a very nice proof of the last inequality for $n=5$ and $k=3$. If you want to see a solution open another topic. This topic I'll close because it's duplicate.
          – Michael Rozenberg
          Sep 9 at 13:09



          這個網誌中的熱門文章

          Why am i infinitely getting the same tweet with the Twitter Search API?

          Is there any way to eliminate the singular point to solve this integral by hand or by approximations?

          Strongly p-embedded subgroups and p-Sylow subgroups.